LSAT and Law School Admissions Forum

Get expert LSAT preparation and law school admissions advice from PowerScore Test Preparation.

User avatar
 Dave Killoran
PowerScore Staff
  • PowerScore Staff
  • Posts: 5852
  • Joined: Mar 25, 2011
|
#94354
Complete Question Explanation
(The complete setup for this game can be found here: lsat/viewtopic.php?f=362&t=3362)

The correct answer choice is (C)

Answer choice (A) is incorrect because all three adults are in boat 1, a violation of the first rule.

Answer choice (B) is incorrect because X and Z are both in boat 2, a violation of the fourth rule.

Answer choice (C) is the correct answer choice.

Answer choice (D) is incorrect because from the third rule, when V is assigned to boat 1, W must be assigned to boat 2.

Answer choice (E) is incorrect because from the contrapositive of the second rule, when G is assigned to boat 1, F must be assigned to boat 1.
 bella243
  • Posts: 65
  • Joined: Apr 29, 2020
|
#76801
Could someone please explain how to solve this question?
 Adam Tyson
PowerScore Staff
  • PowerScore Staff
  • Posts: 5153
  • Joined: Apr 14, 2011
|
#76948
It's a List question, bella243, so attack it using the rules, one by one, keeping in mind that any variable not listed in an answer choice must, by default, be in Boat 2. I would go through it like this:

First rule - there must be an adult in each boat. Check the answers for at least one but no more than two of FGH. Answer A has all three of them, meaning Boat 2 has no adult in it, and that is a loser. Cross it out.

Second rule - if F is in 2, G is in 2. So if I do not see F, meaning that F is in 2, I better not see G either. Answer E violates that - F is not in Boat 1, so it must be in Boat 2, so G shouldn't be here in Boat 1. Cross out E.

Third rule: If V is in 1, W is in 2, so those two cannot both be in Boat 1 together. Answer D has them both in Boat 1, so that's a loser and needs to be crossed out.

Final rule - X and Z have to be kept apart. That means that exactly one of them has to be in Boat 1, with the other in Boat 2. If I see both, or if I see neither, in Boat 1, that answer has to go (seeing neither means they are both in Boat 2 together, which is not allowed). Answer B has neither of them, so it's out, and by process of elimination answer C is the winner!

Get the most out of your LSAT Prep Plus subscription.

Analyze and track your performance with our Testing and Analytics Package.